A measurement was made of the magnetic field due to a tornado, and the result was 13.00 nT to the north. The measurement was made at a position 8.90 km west of the tornado. What was the magnitude (in A) and direction of the current in the funnel of the tornado? Assume the vortex was a long, straight wire carrying a current.

Answers

Answer 1

Answer:

4

Explanation:


Related Questions

A object of mass 3.00 kg is subject to a force Fx that varies with position as in the figure below. A coordinate plane has a horizontal axis labeled x (m) and a vertical axis labeled Fx (N). There are three line segments. The first segment runs from the origin to (4,3). The second segment runs from (4,3) to (11,3). The third segment runs from (11,3) to (17,0). (a) Find the work done by the force on the object as it moves from x = 0 to x = 4.00 m. J (b) Find the work done by the force on the object as it moves from x = 4.00 m to x = 11.0 m. J (c) Find the work done by the force on the object as it moves from x = 11.0 m to x = 17.0 m. J (d) If the object has a speed of 0.450 m/s at x = 0, find its speed at x = 4.00 m and its speed at x = 17.0 m.

Answers

Answer:

Explanation:

An impulse results in a change of momentum.

The impulse is the product of a force and a distance. This will be represented by the area under the curve

a) W = ½(4.00)(3.00) = 6.00 J

b) W = (11.0 - 4.00)(3.00) = 21.0 J

c) W = ½(17.0 - 11.0)(3.00) = 9.00 J

d) ASSUMING the speed at x = 0 is in the direction of applied force

½(3.00)(v₄²) = ½(3.00)(0.450²) + 6.00

v₄ = 2.05 m/s

½(3.00)(v₁₇²) = ½(3.00)(0.450²) + 6.00 + 21.0 + 9.00

v₁₇ = 4.92 m/s

If the initial speed is NOT in the direction of applied force, the final speed will be slightly less in both cases.

Cuando el pistón tiene un volumen de 2x10^-4 m^3, el gas en el pistón está a una presión de 150 kPa. El área del pistón es 0.00133 m^2. Calcular la fuerza que el gas ejerce sobre el embolo del pistón.

Answers

Answer:

F = 1.128 10⁸ Pa

Explanation:

Pressure is defined by

         P = F / A

If the gas is ideal for equal force eds on all the walls, so on the piston area we have

        F = P A

We reduce the pressure to the SI system

       P = 150 kpa (1000 Pa / 1kPa = 150 103 Pa

we calculate

       F = 150 10³ / 0.00133

       F = 1.128 10⁸ Pa

A space ship has four thrusters positioned on the top and bottom, and left and right as shown below. The thrusters can be operated independently or together to help the ship navigate in all directions.
Initially, the Space Probe is floating towards the East, as shown below, with a velocity, v. The pilot then turns on thruster #2.

Select one:

a.
Space ship will have a velocity to the West and will be speeding up.

b.
Space ship will have a velocity to the East and will be speeding up.

c.
Space ship will have a velocity to the East and will be slowing down.

d.
Space ship will have a velocity to the West and will be slowing down.

e.
Ship experiences no change in motion.

Answers

Answer:

The correct answer is - c.  Spaceship will have a velocity to the East and will be slowing down.

Explanation:

In this case, if turned on thruster #2 then it will exert force on the west side as thruster 2 is on the east side and it can be understood by Newton's third law that says each action has the same but opposite reaction.

As the spaceship engine applies force on the east side then according to the law the exhauster gas applies on towards west direction. It will try to decrease the velocity of the spaceship however, the direction of floating still be east side initally.

Q)what are convex mirrors?​

Answers

Answer:

A curved mirror is a mirror with a curved reflecting surface. The surface may be either convex or concave. Most curved mirrors have surfaces that are shaped like part of a sphere, but other shapes are sometimes used in optical devices.

A convex mirror (or lens) is one constructed so that it is thicker in the middle than it is at the edge.

calculate the length of wire.

Answers

Answer:

L = 169.5 m

Explanation:

Using Ohm's Law:

V = IR

where,

V = Voltage = 1.5 V

I = Current = 10 mA = 0.01 A

R = Resistance = ?

Therefore,

1.5 V = (0.01 A)R

R = 150 Ω

But the resistance of a wire is given by the following formula:

[tex]R = \frac{\rho L}{A}[/tex]

where,

ρ = resistivity = 1 x 10⁻⁶ Ω.m

L = length of wire = ?

A = cross-sectional area of wire = πr² = π(0.6 mm)² = π(0.6 x 10⁻³ m)²

A = 1.13 x 10⁻⁶ m²

Therefore,

[tex]150\ \Omega = \frac{(1\ x\ 10^{-6}\ \Omega .m)L}{1.13\ x\ 10^{-6}\ m^2}\\\\L = \frac{150\ \Omega(1.13\ x\ 10^{-6}\ m^2)}{1\ x\ 10^{-6}\ \Omega .m}\\\\[/tex]

L = 169.5 m

Suppose a 60-turn coil lies in the plane of the page in a uniform magnetic field that is directed out of the page. The coil originally has an area of 0.325 m2. It is stretched to have no area in 0.100 s. What is the magnitude (in V) and direction (as seen from above) of the average induced emf if the uniform magnetic field has a strength of 1.60 T

Answers

Answer:

 emf = 312 V

Explanation:

In this exercise the electromotive force is asked, for which we must use Faraday's law

           emf =  [tex]- N \frac{d \Phi }{dt}[/tex]- N dfi / dt

           Ф = B. A = B A cos θ

bold type indicates vectors.

They indicate that the magnetic field is constant, the angle between the normal to the area and the magnetic field is parallel by local cosine values ​​1

It also indicates that the area is reduced from  a₀ = 0.325 me² to a_f = 0 in a time interval of ΔT = 0.100 s, suppose that this reduction is linear

            emf = -N B [tex]\frac{dA}{dT}[/tex]

            emf = - N B (A_f - A₀) / Dt

we calculate

           emf = - 60 1.60 (0 - 0.325) /0.100

           emf = 312 V

The direction of this voltage is exiting the page

A friend lends you the eyepiece of his microscope to use on your own microscope. He claims that since his eyepiece has the same diameter as yours but twice the focal length, the resolving power of your microscope will be doubled. Is his claim valid? Explain.

Answers

Answer:

The resolving power remains same.

Explanation:

The resolving power of the lens is directly proportional to the diameter of the lens not on the focal length.

As the diameter is same but the focal length is doubled so the resolving power remains same.

The cation that is reabsorbed from the urine in response to aldosterone

Answers

Answer:

If decreased blood pressure is detected, the adrenal gland is stimulated by these stretch receptors to release aldosterone, which increases sodium reabsorption from the urine, sweat, and the gut. This causes increased osmolarity in the extracellular fluid, which will eventually return blood pressure toward normal.

5. Steve is driving in his car to take care of some errands. The first errand has him driving to a location 2 km East and 6 km North of his starting location. Once he completes that errand, he drives to the second one which is 4 km East and 2 km South of the first errand. What is the magnitude of the vector that describes how far the car has traveled from its starting point, rounded to the nearest km?​

Answers

Answer:

gshshs

Explanation:

hshsksksksbsbbshd

A 1.5kg block slides along a frictionless surface at 1.3m/s . A second block, sliding at a faster 4.3m/s , collides with the first from behind and sticks to it. The final velocity of the combined blocks is 2.0m/s . What was the mass of the second block?

Answers

Answer:

The mass of the second block=0.457 kg

Explanation:

We are given that

m1=1.5 kg

v1=1.3m/s

v2=4.3 m/s

V=2.0 m/s

We have to find the mass of the second block.

[tex]m_1v_1+m_2v_2=(m_1+m_2)V[/tex]

Let m2=m

Substitute the values

[tex]1.5(1.3)+m(4.3)=(1.5+m)(2)[/tex]

[tex]1.95+4.3m=3+2m[/tex]

[tex]4.3m-2m=3-1.95[/tex]

[tex]2.3m=1.05[/tex]

[tex]m=\frac{1.05}{2.3}[/tex]

[tex]m=0.457 kg[/tex]

Hence,  the mass of the second block=0.457 kg

Why is it advised not to hold the thermometer by its bulb while reading it?

Answers

Because your body heat might change the temperature

. A ball of mass 0.50 kg is rolling across a table top with a speed of 5.0 m/s. When the ball reaches the edge of the table, it rolls down an incline onto the floor 1.0 meter below (without bouncing). What is the speed of the ball when it reaches the floor?

Answers

Answer:

4

Explanation:

Rachel has good distant vision but has a touch of presbyopia. Her near point is 0.60 m. Part A When she wears 2.0 D reading glasses, what is her near point

Answers

Answer:

The right answer is "0.273 m".

Explanation:

Given:

Power (P),

[tex]\frac{1}{f} = 2D[/tex]

Near point,

u = 0.6 m

As we know,

⇒ [tex]\frac{1}{v} -\frac{1}{u}=\frac{1}{f} = 2[/tex]

By substituting the values, we get

⇒ [tex]\frac{1}{v} -\frac{1}{0.6} =2[/tex]

            [tex]\frac{1}{v}=2+\frac{1}{0.6}[/tex]

            [tex]\frac{1}{v} =\frac{1.2+1}{0.6}[/tex]

            [tex]\frac{1}{v}=\frac{2.2}{0.6}[/tex]    

By applying cross-multiplication, we get

          [tex]0.6=2.2 \ v[/tex]

            [tex]v = \frac{0.6}{2.2}[/tex]

      [tex]S_{near} = 0.273 \ m[/tex]

the plane of a 5.0 cm by 8.0 cm rectangular loop wire is parallel to a 0.19 t magnetic field. if the loop carries a current of 6.2 amps, what is the magnitude of the torque on the loop

Answers

here is the state ment :0.2 loop

Why we use semiconductor instead of metal in thermopile.

Answers

Answer:

Semiconductors are not normal materials. They have special properties which conductors/metals cannot exhibit. The main reason for the behavior of semiconductors is that they have paired charge carriers-the electron-hole pair. This is not available in metals.

A load of 25 kg is applied to the lower end and of a steal wire of length 25 m and thickness 3.0mm .The other end of wire is suspeded from a rigid support calculate strain and stress produced in the wire​

Answers

Answer:

the weight of the wire + 25kg

Explanation:

A block with a mass of 0.26 kg is attached to a horizontal spring. The block is pulled back from its equilibrium position until the spring exerts a force of 1.2 N on the block. When the block is released, it oscillates with a frequency of 1.4 Hz. How far was the block pulled back before being released?

Answers

Answer:

2

Explanation:

pulling force because of it force

Answer:

5.9 cm

Explanation:

f: frequency of oscillation

frequency of oscillationk: spring constant

frequency of oscillationk: spring constantm: the mass

[tex]f = \frac{1}{2\pi} \sqrt{ \frac{k}{m} } [/tex]

in this problem we know,

F= 1.4 Hz

m= 0.26 kg

By re-arranging the formula we get

[tex]k = {(2\pi \: f )}^{2} m = {(2\pi(1.4hz))}^{2} 0.26kg = 20.1 \frac{n}{m} [/tex]

The restoring force of the spring is:

F= kx

where

F= 1.2 N

k= 20.1 N/m

x: the displacement of the block

[tex]x = \frac{f}{k} = \frac{1.2 \: n}{20.1 \frac{n}{m} } = 0.059m \: = 5.9 \: cm[/tex]

the rate of cooling determines ....... and ......​

Answers

Answer:

freezing point and melting point

Determine the acceleration of a pendulum bob as it passes through an angle of 15 degrees to the right of the equilibrium point.

Answers

Answer:

Explanation:

Since energy is conserved:

2

mu  

2

 

=  

2

mv  

2

 

+mgh

⇒u  

2

=v  

2

+2gh

⇒(3)  

2

=v  

2

+2(9.8)(0.5−0.5cos60)

⇒v=2m/s

Acceleration of the simple pendulum is 2.62 m/s².

What is meant by a simple pendulum ?

When a point mass is suspended from a fixed support by a light, non-extensible string, the instrument is said to be a simple pendulum.

Here,

Let the mass of the bob be m. The simple pendulum is attached to the fixed support with a string having length l. The pendulum makes an angle of 15° with the vertical from the equilibrium point.

Let T be the tension acting on the string.

As, the bob passes through the angle,

The weight of the bob becomes equal to the vertical component of the tension.

mg = T cos15°

Also, the horizontal component of the tension,

T sin15° = ma

By solving these two equations, we get that,

Acceleration of the simple pendulum,

a = g tan15°

a = 9.8 x 0.267

a = 2.62 m/s²

Hence,

Acceleration of the simple pendulum is 2.62 m/s².

To learn more about simple pendulum, click:

https://brainly.com/question/31816771

#SPJ3

i don't understand this, can someone help please?? ​

Answers

Explanation:

N2 + H2 --> NH3

balance them:

N2 + 3 H2 --> 2 NH3

so if 6 moles of N2 react, 12 moles of NH3 will form.

(you have to look at the big number in front, in this case its N2 and 2 NH3, therefore the amount of N2 will produce double the amount of NH3 )

Two forces are acting on a body. One acts east, the other at 35° north of east. If the
two forces are equal in magnitude of 50 N, find the resultant using the Law of Sines
and the Law of Cosines. Please answer with full solution. Thanks

Answers

A=B=50NAngle=theta=35°

We know

[tex]\boxed{\sf R=\sqrt{A^2+B^2+2ABcos\Theta}}[/tex]

[tex]\\ \sf\longmapsto R=\sqrt{50^2+50^2+2(50)(50)cos35}[/tex]

[tex]\\ \sf\longmapsto R=\sqrt{2500+2500+2(2500)\times (-0.9)}[/tex]

[tex]\\ \sf\longmapsto R=\sqrt{5000+5000(-0.9)}[/tex]

[tex]\\ \sf\longmapsto R=\sqrt{5000+(-4500)}[/tex]

[tex]\\ \sf\longmapsto R=\sqrt{5000-4500}[/tex]

[tex]\\ \sf\longmapsto R=\sqrt{-500}[/tex]

[tex]\\ \sf\longmapsto R=22.4i[/tex]

Resultant using the Law of Sines and the Law of Cosines will be R=95 N

What is force?

Force is an external agent applied on any object to displace it from its position. Force is a vector quantity, so with magnitude it also requires direction. Direction is necessary to examine the effect of the force and to find the equilibrium of the force.

The Magnitude of two forces =50 N

Angle between the forces = 35

By using the resultant formula

[tex]\rm R=\sqrt{A^2+B^2+2ABCos\theta}[/tex]

[tex]\rm R=\sqrt{50^2+50^2+2(50)(50)Cos35}[/tex]

[tex]\rm R=\sqrt{5000+5000(0.81)}[/tex]

[tex]\rm R=\sqrt{5000+4500}[/tex]

[tex]\rm R=95\ N[/tex]

Hence the Resultant using the Law of Sines and the Law of Cosines will be R=95 N

To know more about force follow

https://brainly.com/question/25239010

The kinetic energy of a particle of mass 500g is 4.8j. Determine the velocity of the particle

Answers

Answer:

4.38 m/s

Explanation:

The answer is 4.38 m/s

What do you understand by moment of inertia and torque?
Word limit 50-60

Please don't copy from any sources. You can rewrite. Plagiarism will be check. Thank you.

Answers

Answer:

Moment of inertia, in physics, quantitative measure of the rotational inertia of a body—i.e., the opposition that the body exhibits to having its speed of rotation about an axis altered by the application of a torque (turning force). The axis may be internal or external and may or may not be fixed.

A runner has a temperature of 40°c and is giving off heat at the rate of 50cal/s (a) What is the rate of heat loss in watts? (b) How long will it take for this person's temperature to return to 37°c if his mass is 90kg.

Answers

Answer:

(a)  209 Watt

(b) 4482.8 seconds

Explanation:

(a) P = 50×4.18

Where P = rate of heat loss in watt

    P = 209 Watt

Applying,

Q = cm(t₁-t₂)................ Equation 1

Where Q = amount of heat given off, c = specific heat capacity capacity of human, m = mass of the person, t₁ and t₂ = initial and final temperature.

From the question,

Given: m = 90 kg, t₁ = 40°C, t₂ = 37°C

Constant: c = 3470 J/kg.K

Substtut these values into equation 1

Q = 90×3470(40-37)

Q = 936900 J

But,

P = Q/t.............. Equation 2

Where t = time

t = Q/P............ Equation 3

Given: P = 209 Watt, Q = 936900

Substitute into equation 3

t = 936900/209

t = 4482.8 seconds

An equation for the period of a planet is 4 pie² r³/Gm where T is in secs, r is in meters, G is in m³/kgs² m is in kg, show that the equation is dimensionally correct.​

Answers

Answer:

[tex]\displaystyle T = \sqrt{\frac{4\, \pi^{2} \, r^{3}}{G \cdot m}}[/tex].

The unit of both sides of this equation are [tex]\rm s[/tex].

Explanation:

The unit of the left-hand side is [tex]\rm s[/tex], same as the unit of [tex]T[/tex].

The following makes use of the fact that for any non-zero value [tex]x[/tex], the power [tex]x^{-1}[/tex] is equivalent to [tex]\displaystyle \frac{1}{x}[/tex].

On the right-hand side of this equation:

[tex]\pi[/tex] has no unit.The unit of [tex]r[/tex] is [tex]\rm m[/tex].The unit of [tex]G[/tex] is [tex]\displaystyle \rm \frac{m^{3}}{kg \cdot s^{2}}[/tex], which is equivalent to [tex]\rm m^{3} \cdot kg^{-1} \cdot s^{-2}[/tex].The unit of [tex]m[/tex] is [tex]\rm kg[/tex].

[tex]\begin{aligned}& \rm \sqrt{\frac{(m)^{3}}{(m^{3} \cdot kg^{-1} \cdot s^{-2}) \cdot (kg)}} \\ &= \rm \sqrt{\frac{m^{3}}{m^{3} \cdot s^{-2}}} = \sqrt{s^{2}} = s\end{aligned}[/tex].

Hence, the unit on the right-hand side of this equation is also [tex]\rm s[/tex].

A long copper wire of radius 0.321 mm has a linear charge density of 0.100 μC/m. Find the electric field at a point 5.00 cm from the center of the wire. (in Nm2/C, keep 3 significant figures)

Answers

Answer:

[tex]E=35921.96N/C[/tex]

Explanation:

From the question we are told that:

Radius [tex]r=0.321mm[/tex]

Charge Density [tex]\mu=0.100[/tex]

Distance [tex]d= 5.00 cm[/tex]

Generally the equation for electric field is mathematically given by

[tex]E=\frac{mu}{2\pi E_0r}[/tex]

[tex]E=\frac{0.100*10^{-6}}{2*3.142*8.86*10^{-12}*5*10^{-2}}[/tex]

[tex]E=35921.96N/C[/tex]

15 . A scientist who studies the whole environment as a working unit .

Botanist
Chemist
Ecologist
Entomologist

Answers

Answer:

Ecologist.

Your answer is Ecologist.

(Ecologist) is a scientist who studies the whole environment as a working unit.

When the drag force on an object falling through the air equals the force of gravity, the object has reached
terminal force.
terminal acceleration,
terminal illness.
terminal velocity

Answers

The answer is terminal force

A 20 N south magnetic force pushes a charged particle traveling with a velocity of 4 m/s west through a 5 T magnetic field pointing downwards . What is the charge of the particle ?

Answers

Answer:

Charge of the particle is 1 coulomb.

Explanation:

Force, F:

[tex]{ \bf{F=BeV}}[/tex]

F is magnetic force.

B is the magnetic flux density.

e is the charge of the particle.

V is the velocity

[tex]{ \sf{20 = (5 \times e \times 4)}} \\ { \sf{20e = 20}} \\ { \sf{e = 1 \: coulomb}}[/tex]

A nearsighted person has a near point of 50 cmcm and a far point of 100 cmcm. Part A What power lens is necessary to correct this person's vision to allow her to see distant objects

Answers

Answer:

P = -1 D

Explanation:

For this exercise we must use the equation of the constructor

       / f = 1 / p + 1 / q

where f is the focal length, p and q is the distance to the object and the image, respectively

The far view point is at p =∞  and its image must be at q = -100 cm = 1 m, the negative sign is because the image is on the same side as the image  

        [tex]\frac{1}{f} = \frac{1}{infinity} + \frac{1}{-1}[/tex]

         f = 1 m

         P = 1/f

          P = -1 D

Other Questions
gegygwugduwbudbwbdubwudbuwh7fh7whf8hw8hf8hw8hf8e Why does the GI tract have a plexus in the muscalaris and nerves in the mucosa? Do a little research on your community's Web sites to find out what kind of conservation or preservation measures have been accomplished and the effect they have had on the community. When you have completed this task, write a 400-word paper that outlines what has been done and what is to be done in the future. Determine the type of quadrilateral given the following coordinates. Show and explain all steps to prove your answer. A(2, 3) B(-1, 4) C(0, 2) D(-3, 3) Refer to the picture above Type the correct answer in the box. Use numerals instead of words. If necessary, use / for the fraction bar.If x+2 is a factor of x^3-6x^2-11x+k then k= ng andSegmene Ruler Postulate to find segment lengths.e the Segment Addition Postulate to find segmcopy segments and compare segments for congfind the length indicated.1.?20T.UV93 Factorize the following by splitting the middle term:-(a) 3x^2 +11x+30 Can you answer this math homework? Please! Evaluate the function.f(x) = 4x + 73 18Find f(-9) Please help. I don't understand how to solve for number 17, 19, and 21. Please show how you solved each problem 2nd time. l'll give ya brainliest if you answer lol anywaycan someone make this sentence more like a 7th grader said it or something:The West has large open areas untouched by people. The land includes large forests, deserts, and mountains. The variety causes wide differences in weather. Hiking and camping are popular. The environment is an important issue. Ranching, lumbering, and mining are important industries. Los Angeles, Seattle, and Denver are major cities Camille is attending a fundraiser. She pays for her admission and buys raffle tickets for $5dollar each. If she buys 10 raffle tickets, then she would spend a total of $135 at the fundraiser.The number S of dollars Camille spends at the fundraiser is a function of r, the number of raffle tickets she buys.Write the function's formula. Which of the following is NOT something you should always do before a journey? A. Map the road ahead B. Deflate and reinflate your tires C. Assess your fitness to drive D. Use the GPS properly, if necessaryHelp plz Find the 20th term of the following sequence.-6, -4,-2, O,... is the answer correct or wrong "Neanderthals were much more like man than thought", if today we meet a Neanderthal face to face: (choose the correct option) *A. I would recognize it because they were taller and stronger than human beings.B. They could only live in Europe and no one would notice.C. I couldn't recognize it because they were so much like us.Choose the correct option. The Stone Age is divided into Neolithic and Paleolithic, then..A. Neanderthals worked stone and some metals.B. Neanderthals lived until the Neolithic because they discovered agriculture.C. Neanderthals had great knowledge of stoneworking techniques.Choose the correct optionA. Neanderthals and humans are the sameB. Between Neanderthals and humans there is a minimal difference.C. Humans were the only ones who made paintings. please help with this too Write the words that represent s + 21. Anyone knows the answer?